ChaseDream

标题: 求助CR JJ 13考古 [打印本页]

作者: sive    时间: 2011-3-4 19:40
标题: 求助CR JJ 13考古
In Gandania, where the government has a monopoly on tobacco sales, the incidence of smoking-related health problems has risen steadily for the last twenty years. The health secretary recently proposed a series of laws aimed at curtailing tobacco use in Gandania. Profits from tobacco sales, however, account for ten percent of Gandania’s annual revenues. Therefore, Gandania cannot afford to institute the proposed laws.

Which of the following, if true, most seriously weakens the argument?

A.    All health care in Gandania is government-funded.
B.    Implementing the proposed laws is not likely to cause a significant increase in the amount of tobacco Gandania exports.
C.    The percentage of revenue Gandania receives from tobacco sales has remained steady in recent years.
D.    rofits from tobacco sales far surpass any other single source of revenue for the Gandanian government.
E.    No government official in Gandania has ever previously proposed laws aimed at curtailing tobacco use.

Weaken,答案是A. 但是如果问题换成了Assumption,答案是否会是B?

JJ14就是Assumption,请指教!

作者: prayerv    时间: 2011-3-4 21:22
我觉得这个考古靠谱…………  
求狗主人确认
作者: Blake1171    时间: 2011-3-4 21:34
话说前两天刚做了这个题来着,等狗主人确定~~~
作者: sdcar2010    时间: 2011-3-4 21:35
B is not a necessary assumption.
作者: sive    时间: 2011-3-4 21:39
我不是狗主人...刚才在考古区看到别人写的,而JJ里面没有.
只是想问,如果问题不是问Weaken,问Assumption的话,那答案还是A么?
作者: sive    时间: 2011-3-4 21:40
而看到RC JJ14主人写道他选的MS是B,有什么Export,莫非题目有变?
作者: sdcar2010    时间: 2011-3-4 21:49
A is not a necessary assumption.
作者: elnys    时间: 2011-3-4 22:03
14和这个不是一题啊。。
作者: Lyssazhao    时间: 2011-3-14 20:38
如果选项不变,ASSUMPTION我觉得D比较合适
作者: backlighting    时间: 2011-3-14 21:32
我同意楼上
然后同等待NN确认答案
作者: 优之菡萏    时间: 2011-3-15 09:08
UP
作者: clair1024    时间: 2011-3-15 15:53
狗狗问的是得出的结论是啥 应该选什么呢 E?
作者: sdcar2010    时间: 2011-3-15 18:56
None of the answer choices is a necessary assumption.

The assumption might be:
If a proposed law would hurt Gandania's annual revenue, the government would not institute said law.
作者: rooneyfang    时间: 2011-3-15 22:13
If the question asks for ASSUMPTION.

C is the correct answer.

The line of reasoning states that (because) tobacco acounts for 10 persent of Gandania's anual revenue, Gandania can not afford to lose the revenue by enact the new regulation.

The underlying assumption is the 10 percent revenue, which constructed by tobacco sales, accounts for Gandania's law making.

C says the revenue percentage from tobacco sales remain steady recent years. Even it means when total revenue increases, tabacco sales increases, and vice versa. However, no matter which is the case. the hard core reality is tobacco sales constitute a pivotal part of Gandania's revenue.




欢迎光临 ChaseDream (https://forum.chasedream.com/) Powered by Discuz! X3.3